Is this expression true?












0












$begingroup$


Let $a_1=b_1/h,...,a_n=b_n/hinmathbb{R}$ with $hinmathbb{R}$ small. It's true that, given a $alphainmathbb{R}$:
begin{eqnarray}
(a_1+...+a_n)^alpha=sum_{i=1}^n (a_i)^alpha+mathcal{O}left(frac{1}{h^2}right)
end{eqnarray}

?? Is there any (good) reference where there is proof of some theorem / lemma that I can use to justify this?










share|cite|improve this question









$endgroup$



migrated from mathoverflow.net Jan 11 at 14:41


This question came from our site for professional mathematicians.


















  • $begingroup$
    This is false if $b_1=cdots =b_n=1$ and $alpha >2$.
    $endgroup$
    – abx
    Jan 5 at 5:21










  • $begingroup$
    You are asking in the wrong forum, I think.
    $endgroup$
    – GEdgar
    Jan 5 at 14:03










  • $begingroup$
    There is a problem when $b_1 = h$, $b_2 = -h$, and $alpha = -1$. I think you're missing some constraints if you want this to be true.
    $endgroup$
    – Eric Towers
    Jan 11 at 15:00
















0












$begingroup$


Let $a_1=b_1/h,...,a_n=b_n/hinmathbb{R}$ with $hinmathbb{R}$ small. It's true that, given a $alphainmathbb{R}$:
begin{eqnarray}
(a_1+...+a_n)^alpha=sum_{i=1}^n (a_i)^alpha+mathcal{O}left(frac{1}{h^2}right)
end{eqnarray}

?? Is there any (good) reference where there is proof of some theorem / lemma that I can use to justify this?










share|cite|improve this question









$endgroup$



migrated from mathoverflow.net Jan 11 at 14:41


This question came from our site for professional mathematicians.


















  • $begingroup$
    This is false if $b_1=cdots =b_n=1$ and $alpha >2$.
    $endgroup$
    – abx
    Jan 5 at 5:21










  • $begingroup$
    You are asking in the wrong forum, I think.
    $endgroup$
    – GEdgar
    Jan 5 at 14:03










  • $begingroup$
    There is a problem when $b_1 = h$, $b_2 = -h$, and $alpha = -1$. I think you're missing some constraints if you want this to be true.
    $endgroup$
    – Eric Towers
    Jan 11 at 15:00














0












0








0





$begingroup$


Let $a_1=b_1/h,...,a_n=b_n/hinmathbb{R}$ with $hinmathbb{R}$ small. It's true that, given a $alphainmathbb{R}$:
begin{eqnarray}
(a_1+...+a_n)^alpha=sum_{i=1}^n (a_i)^alpha+mathcal{O}left(frac{1}{h^2}right)
end{eqnarray}

?? Is there any (good) reference where there is proof of some theorem / lemma that I can use to justify this?










share|cite|improve this question









$endgroup$




Let $a_1=b_1/h,...,a_n=b_n/hinmathbb{R}$ with $hinmathbb{R}$ small. It's true that, given a $alphainmathbb{R}$:
begin{eqnarray}
(a_1+...+a_n)^alpha=sum_{i=1}^n (a_i)^alpha+mathcal{O}left(frac{1}{h^2}right)
end{eqnarray}

?? Is there any (good) reference where there is proof of some theorem / lemma that I can use to justify this?







reference-request real-analysis






share|cite|improve this question













share|cite|improve this question











share|cite|improve this question




share|cite|improve this question










asked Jan 4 at 23:20









Leonardo S. VieiraLeonardo S. Vieira

11




11




migrated from mathoverflow.net Jan 11 at 14:41


This question came from our site for professional mathematicians.









migrated from mathoverflow.net Jan 11 at 14:41


This question came from our site for professional mathematicians.














  • $begingroup$
    This is false if $b_1=cdots =b_n=1$ and $alpha >2$.
    $endgroup$
    – abx
    Jan 5 at 5:21










  • $begingroup$
    You are asking in the wrong forum, I think.
    $endgroup$
    – GEdgar
    Jan 5 at 14:03










  • $begingroup$
    There is a problem when $b_1 = h$, $b_2 = -h$, and $alpha = -1$. I think you're missing some constraints if you want this to be true.
    $endgroup$
    – Eric Towers
    Jan 11 at 15:00


















  • $begingroup$
    This is false if $b_1=cdots =b_n=1$ and $alpha >2$.
    $endgroup$
    – abx
    Jan 5 at 5:21










  • $begingroup$
    You are asking in the wrong forum, I think.
    $endgroup$
    – GEdgar
    Jan 5 at 14:03










  • $begingroup$
    There is a problem when $b_1 = h$, $b_2 = -h$, and $alpha = -1$. I think you're missing some constraints if you want this to be true.
    $endgroup$
    – Eric Towers
    Jan 11 at 15:00
















$begingroup$
This is false if $b_1=cdots =b_n=1$ and $alpha >2$.
$endgroup$
– abx
Jan 5 at 5:21




$begingroup$
This is false if $b_1=cdots =b_n=1$ and $alpha >2$.
$endgroup$
– abx
Jan 5 at 5:21












$begingroup$
You are asking in the wrong forum, I think.
$endgroup$
– GEdgar
Jan 5 at 14:03




$begingroup$
You are asking in the wrong forum, I think.
$endgroup$
– GEdgar
Jan 5 at 14:03












$begingroup$
There is a problem when $b_1 = h$, $b_2 = -h$, and $alpha = -1$. I think you're missing some constraints if you want this to be true.
$endgroup$
– Eric Towers
Jan 11 at 15:00




$begingroup$
There is a problem when $b_1 = h$, $b_2 = -h$, and $alpha = -1$. I think you're missing some constraints if you want this to be true.
$endgroup$
– Eric Towers
Jan 11 at 15:00










0






active

oldest

votes











Your Answer





StackExchange.ifUsing("editor", function () {
return StackExchange.using("mathjaxEditing", function () {
StackExchange.MarkdownEditor.creationCallbacks.add(function (editor, postfix) {
StackExchange.mathjaxEditing.prepareWmdForMathJax(editor, postfix, [["$", "$"], ["\\(","\\)"]]);
});
});
}, "mathjax-editing");

StackExchange.ready(function() {
var channelOptions = {
tags: "".split(" "),
id: "69"
};
initTagRenderer("".split(" "), "".split(" "), channelOptions);

StackExchange.using("externalEditor", function() {
// Have to fire editor after snippets, if snippets enabled
if (StackExchange.settings.snippets.snippetsEnabled) {
StackExchange.using("snippets", function() {
createEditor();
});
}
else {
createEditor();
}
});

function createEditor() {
StackExchange.prepareEditor({
heartbeatType: 'answer',
autoActivateHeartbeat: false,
convertImagesToLinks: true,
noModals: true,
showLowRepImageUploadWarning: true,
reputationToPostImages: 10,
bindNavPrevention: true,
postfix: "",
imageUploader: {
brandingHtml: "Powered by u003ca class="icon-imgur-white" href="https://imgur.com/"u003eu003c/au003e",
contentPolicyHtml: "User contributions licensed under u003ca href="https://creativecommons.org/licenses/by-sa/3.0/"u003ecc by-sa 3.0 with attribution requiredu003c/au003e u003ca href="https://stackoverflow.com/legal/content-policy"u003e(content policy)u003c/au003e",
allowUrls: true
},
noCode: true, onDemand: true,
discardSelector: ".discard-answer"
,immediatelyShowMarkdownHelp:true
});


}
});














draft saved

draft discarded


















StackExchange.ready(
function () {
StackExchange.openid.initPostLogin('.new-post-login', 'https%3a%2f%2fmath.stackexchange.com%2fquestions%2f3069910%2fis-this-expression-true%23new-answer', 'question_page');
}
);

Post as a guest















Required, but never shown

























0






active

oldest

votes








0






active

oldest

votes









active

oldest

votes






active

oldest

votes
















draft saved

draft discarded




















































Thanks for contributing an answer to Mathematics Stack Exchange!


  • Please be sure to answer the question. Provide details and share your research!

But avoid



  • Asking for help, clarification, or responding to other answers.

  • Making statements based on opinion; back them up with references or personal experience.


Use MathJax to format equations. MathJax reference.


To learn more, see our tips on writing great answers.




draft saved


draft discarded














StackExchange.ready(
function () {
StackExchange.openid.initPostLogin('.new-post-login', 'https%3a%2f%2fmath.stackexchange.com%2fquestions%2f3069910%2fis-this-expression-true%23new-answer', 'question_page');
}
);

Post as a guest















Required, but never shown





















































Required, but never shown














Required, but never shown












Required, but never shown







Required, but never shown

































Required, but never shown














Required, but never shown












Required, but never shown







Required, but never shown







Popular posts from this blog

Mario Kart Wii

What does “Dominus providebit” mean?

Antonio Litta Visconti Arese